When does the tensor product distribute over an infinite direct product?












2














It is well known that the tensor product of $R$-modules over some ring $R$ does not, in general, distribute over infinite direct products, an obvious example being $mathbb Z_p otimes_mathbb Z mathbb Q neq 0$. I also know that a sufficient condition for the tensor product to distribute is this.



What other sufficient conditions are there? For instance, it would seem intuitive that $mathbb Q otimes_mathbb Z prod_mathbb N mathbb Z cong prod_mathbb N mathbb Q$. But is it?










share|cite|improve this question




















  • 1




    Are you considering these as groups? Please edit the question to be more specific.
    – Shaun
    Dec 27 '18 at 3:54






  • 1




    @Shaun Thanks for your comment. The isomorphism should be one of $mathbb Z$-modules, in the example at the end. I'll edit accordingly.
    – foaly
    Dec 27 '18 at 4:01
















2














It is well known that the tensor product of $R$-modules over some ring $R$ does not, in general, distribute over infinite direct products, an obvious example being $mathbb Z_p otimes_mathbb Z mathbb Q neq 0$. I also know that a sufficient condition for the tensor product to distribute is this.



What other sufficient conditions are there? For instance, it would seem intuitive that $mathbb Q otimes_mathbb Z prod_mathbb N mathbb Z cong prod_mathbb N mathbb Q$. But is it?










share|cite|improve this question




















  • 1




    Are you considering these as groups? Please edit the question to be more specific.
    – Shaun
    Dec 27 '18 at 3:54






  • 1




    @Shaun Thanks for your comment. The isomorphism should be one of $mathbb Z$-modules, in the example at the end. I'll edit accordingly.
    – foaly
    Dec 27 '18 at 4:01














2












2








2


2





It is well known that the tensor product of $R$-modules over some ring $R$ does not, in general, distribute over infinite direct products, an obvious example being $mathbb Z_p otimes_mathbb Z mathbb Q neq 0$. I also know that a sufficient condition for the tensor product to distribute is this.



What other sufficient conditions are there? For instance, it would seem intuitive that $mathbb Q otimes_mathbb Z prod_mathbb N mathbb Z cong prod_mathbb N mathbb Q$. But is it?










share|cite|improve this question















It is well known that the tensor product of $R$-modules over some ring $R$ does not, in general, distribute over infinite direct products, an obvious example being $mathbb Z_p otimes_mathbb Z mathbb Q neq 0$. I also know that a sufficient condition for the tensor product to distribute is this.



What other sufficient conditions are there? For instance, it would seem intuitive that $mathbb Q otimes_mathbb Z prod_mathbb N mathbb Z cong prod_mathbb N mathbb Q$. But is it?







abstract-algebra modules tensor-products infinite-product






share|cite|improve this question















share|cite|improve this question













share|cite|improve this question




share|cite|improve this question








edited Dec 27 '18 at 22:36









user26857

39.2k123983




39.2k123983










asked Dec 27 '18 at 3:36









foaly

474515




474515








  • 1




    Are you considering these as groups? Please edit the question to be more specific.
    – Shaun
    Dec 27 '18 at 3:54






  • 1




    @Shaun Thanks for your comment. The isomorphism should be one of $mathbb Z$-modules, in the example at the end. I'll edit accordingly.
    – foaly
    Dec 27 '18 at 4:01














  • 1




    Are you considering these as groups? Please edit the question to be more specific.
    – Shaun
    Dec 27 '18 at 3:54






  • 1




    @Shaun Thanks for your comment. The isomorphism should be one of $mathbb Z$-modules, in the example at the end. I'll edit accordingly.
    – foaly
    Dec 27 '18 at 4:01








1




1




Are you considering these as groups? Please edit the question to be more specific.
– Shaun
Dec 27 '18 at 3:54




Are you considering these as groups? Please edit the question to be more specific.
– Shaun
Dec 27 '18 at 3:54




1




1




@Shaun Thanks for your comment. The isomorphism should be one of $mathbb Z$-modules, in the example at the end. I'll edit accordingly.
– foaly
Dec 27 '18 at 4:01




@Shaun Thanks for your comment. The isomorphism should be one of $mathbb Z$-modules, in the example at the end. I'll edit accordingly.
– foaly
Dec 27 '18 at 4:01










2 Answers
2






active

oldest

votes


















8














Here is a general criterion. (All tensor products in this answer are over $R$.)




Theorem: Let $R$ be a ring and let $M$ be a right $R$-module. Then the following are equivalent.




  1. The functor $Motimes -$ preserves products: that is, for every family $(A_i)$ of left $R$-modules, the canonical map $Motimesprod A_ito prod Motimes A_i$ is an isomorphism.


  2. $M$ is finitely presented.





Proof: First, suppose $M$ has a finite presentation $$R^mto R^nto Mto 0$$ and let $(A_i)$ be any family of left $R$-modules. We then get a commutative diagram
$$require{AMScd}
begin{CD}
R^motimes prod A_i @>>> R^notimes prod A_i @>>> Motimes prod A_i @>>> 0\
@VV{}V @VV{}V @VV{}V \
prod R^motimes A_i @>>> prod R^notimes A_i @>>> prod Motimes A_i @>>> 0
end{CD}$$
whose rows are exact. Now note that $R^motimes prod A_icong (prod A_i)^m$ and $prod R^motimes A_icong prod A_i^m$, and our vertical map between them is easily seen to be the canonical isomorphism which interchanges the products. So the left vertical map is an isomorphism and similarly so is the middle vertical map. By the five lemma, it follows that the right vertical map is an isomorphism, and thus $Motimes -$ preserves products.



Conversely, suppose $Motimes-$ preserves products. In particular, then, the canonical map $$varphi: Motimes R^Mto M^M$$ is an isomorphism. Considering the identity map $id:Mto M$ as an element of the product $M^M$, we have $$id=varphi(sum m_i otimes f_i)$$ for some finite collection of elements $m_iin M$ and $f_i:Mto R$. Evaluating both sides of this equation at an element $min M$ we find $$m=sum m_if_i(m).$$ Thus every element of $M$ is a linear combination of the elements $m_i$, so $M$ is finitely generated.



Now let $$0to Kto Fto Mto 0$$ be a presentation of $M$ with $F$ a finitely generated free module. To conclude $M$ is finitely presented, we must show that $K$ is finitely generated. Now let $(A_i)$ be any family of flat left $R$-modules and consider the commutative diagram
$$require{AMScd}
begin{CD}
Kotimes prod A_i @>>> Fotimes prod A_i @>>> Motimes prod A_i @>>> 0\
@VV{}V @VV{}V @VV{}V \
prod Kotimes A_i @>{alpha}>> prod Fotimes A_i @>>> prod Motimes A_i @>>> 0
end{CD}$$

which again has exact rows. The right vertical map is an isomorphism by hypothesis and the middle vertical map is an isomorphism since $F$ is finitely generated and free. Moreover, the map $alpha$ is injective since the $A_i$ are flat. A simple diagram chase now shows that the left vertical map is surjective.



Thus the canonical map $Kotimes prod A_ito prod Kotimes A_i$ is surjective for any family of flat modules $(A_i)$. In particular, the canonical map $Kotimes R^Kto K^K$ is surjective, which we have seen above implies that $K$ is finitely generated.





By similar arguments, you can show that the canonical map $Motimes prod A_itoprod Motimes A_i$ is always a surjection iff $M$ is finitely generated. Indeed, the forward direction is already contained in the forward direction of the proof above, and the reverse direction is similar to the proof of the reverse direction above.






share|cite|improve this answer























  • Dear Eric Wofsey, thank you so much for this wonderful answer. I have a follow-up question: are there conditions on the $A_i$ that loosen the requirement on $M$? For instance $mathbb Q otimes_mathbb Z prod mathbb Z / p^i mathbb Z$ clearly does not distribute, while in some sense -- as you commented on Qiaochu Yuan's answer -- $mathbb Q otimes_mathbb Z prod mathbb Z$ does.
    – foaly
    Dec 28 '18 at 6:13












  • First of all, you absolutely should not think that $mathbb Q otimes_mathbb Z prod mathbb Z$ "distributes" in any reasonable sense. Yes, it is isomorphic to what you would get by distributing, but the isomorphism is highly noncanonical and probably can't even be proved to exist without the axiom of choice. It's essentially a total accident that they are isomorphic.
    – Eric Wofsey
    Dec 28 '18 at 6:53












  • Second, notice that the counterexamples constructed in my proof only ever actually use the case where $A_i=R$ for all $i$. What could be nicer than that? So, I doubt there is any interesting restriction you could put on the $A_i$ that would weaken the condition on $M$.
    – Eric Wofsey
    Dec 28 '18 at 6:55



















4















For instance, it would seem intuitive that $mathbb Q otimes_mathbb Z prod_mathbb N mathbb Z cong prod_mathbb N mathbb Q$. But is it?




It is not. The natural map from the LHS to the RHS fails to be surjective; its image is the subgroup of $prod mathbb{Q}$ consisting of those sequences whose terms have a common denominator, and hence does not include, for example, $prod frac{1}{n}$.



In general, for modules over a commutative ring $k$, the tensor product $M otimes_k (-)$ preserves infinite products if $M$ is finitely presented projective. I had thought at some point about whether it suffices for $M$ just to be finitely presented, but I don't remember what the conclusion was off the top of my head.






share|cite|improve this answer

















  • 4




    To be pedantic, those groups actually are isomorphic (though not by the natural map), since both are $mathbb{Q}$-vector spaces of dimension $mathfrak{c}$. Of course, the right question is whether the natural map is an isomorphism.
    – Eric Wofsey
    Dec 27 '18 at 5:11












  • Thank you for this answer, Qiaochu Yuan. Please allow me to ask some naive questions. As you point out, the canonical map is not an isomorphism. Nevertheless, as @EricWofsey points out, the two modules have some isomorphism. In what sense is the question after the natural map "the right question"? Also I am not familiar with the notation "dimension $mathfrak c$"; I assume it means countably infinite.
    – foaly
    Dec 28 '18 at 6:20












  • @foaly: "tensor product distributes over infinite direct products" is a question about the natural map being an isomorphism. Asking for there to be some arbitrary isomorphism is basically never the question you actually care about in practice.
    – Qiaochu Yuan
    Dec 28 '18 at 9:25










  • @QiaochuYuan it does not seem obvious to me why one would not be interested in the existence of some isomorphism. Can you elaborate?
    – foaly
    Dec 28 '18 at 9:29











Your Answer





StackExchange.ifUsing("editor", function () {
return StackExchange.using("mathjaxEditing", function () {
StackExchange.MarkdownEditor.creationCallbacks.add(function (editor, postfix) {
StackExchange.mathjaxEditing.prepareWmdForMathJax(editor, postfix, [["$", "$"], ["\\(","\\)"]]);
});
});
}, "mathjax-editing");

StackExchange.ready(function() {
var channelOptions = {
tags: "".split(" "),
id: "69"
};
initTagRenderer("".split(" "), "".split(" "), channelOptions);

StackExchange.using("externalEditor", function() {
// Have to fire editor after snippets, if snippets enabled
if (StackExchange.settings.snippets.snippetsEnabled) {
StackExchange.using("snippets", function() {
createEditor();
});
}
else {
createEditor();
}
});

function createEditor() {
StackExchange.prepareEditor({
heartbeatType: 'answer',
autoActivateHeartbeat: false,
convertImagesToLinks: true,
noModals: true,
showLowRepImageUploadWarning: true,
reputationToPostImages: 10,
bindNavPrevention: true,
postfix: "",
imageUploader: {
brandingHtml: "Powered by u003ca class="icon-imgur-white" href="https://imgur.com/"u003eu003c/au003e",
contentPolicyHtml: "User contributions licensed under u003ca href="https://creativecommons.org/licenses/by-sa/3.0/"u003ecc by-sa 3.0 with attribution requiredu003c/au003e u003ca href="https://stackoverflow.com/legal/content-policy"u003e(content policy)u003c/au003e",
allowUrls: true
},
noCode: true, onDemand: true,
discardSelector: ".discard-answer"
,immediatelyShowMarkdownHelp:true
});


}
});














draft saved

draft discarded


















StackExchange.ready(
function () {
StackExchange.openid.initPostLogin('.new-post-login', 'https%3a%2f%2fmath.stackexchange.com%2fquestions%2f3053569%2fwhen-does-the-tensor-product-distribute-over-an-infinite-direct-product%23new-answer', 'question_page');
}
);

Post as a guest















Required, but never shown

























2 Answers
2






active

oldest

votes








2 Answers
2






active

oldest

votes









active

oldest

votes






active

oldest

votes









8














Here is a general criterion. (All tensor products in this answer are over $R$.)




Theorem: Let $R$ be a ring and let $M$ be a right $R$-module. Then the following are equivalent.




  1. The functor $Motimes -$ preserves products: that is, for every family $(A_i)$ of left $R$-modules, the canonical map $Motimesprod A_ito prod Motimes A_i$ is an isomorphism.


  2. $M$ is finitely presented.





Proof: First, suppose $M$ has a finite presentation $$R^mto R^nto Mto 0$$ and let $(A_i)$ be any family of left $R$-modules. We then get a commutative diagram
$$require{AMScd}
begin{CD}
R^motimes prod A_i @>>> R^notimes prod A_i @>>> Motimes prod A_i @>>> 0\
@VV{}V @VV{}V @VV{}V \
prod R^motimes A_i @>>> prod R^notimes A_i @>>> prod Motimes A_i @>>> 0
end{CD}$$
whose rows are exact. Now note that $R^motimes prod A_icong (prod A_i)^m$ and $prod R^motimes A_icong prod A_i^m$, and our vertical map between them is easily seen to be the canonical isomorphism which interchanges the products. So the left vertical map is an isomorphism and similarly so is the middle vertical map. By the five lemma, it follows that the right vertical map is an isomorphism, and thus $Motimes -$ preserves products.



Conversely, suppose $Motimes-$ preserves products. In particular, then, the canonical map $$varphi: Motimes R^Mto M^M$$ is an isomorphism. Considering the identity map $id:Mto M$ as an element of the product $M^M$, we have $$id=varphi(sum m_i otimes f_i)$$ for some finite collection of elements $m_iin M$ and $f_i:Mto R$. Evaluating both sides of this equation at an element $min M$ we find $$m=sum m_if_i(m).$$ Thus every element of $M$ is a linear combination of the elements $m_i$, so $M$ is finitely generated.



Now let $$0to Kto Fto Mto 0$$ be a presentation of $M$ with $F$ a finitely generated free module. To conclude $M$ is finitely presented, we must show that $K$ is finitely generated. Now let $(A_i)$ be any family of flat left $R$-modules and consider the commutative diagram
$$require{AMScd}
begin{CD}
Kotimes prod A_i @>>> Fotimes prod A_i @>>> Motimes prod A_i @>>> 0\
@VV{}V @VV{}V @VV{}V \
prod Kotimes A_i @>{alpha}>> prod Fotimes A_i @>>> prod Motimes A_i @>>> 0
end{CD}$$

which again has exact rows. The right vertical map is an isomorphism by hypothesis and the middle vertical map is an isomorphism since $F$ is finitely generated and free. Moreover, the map $alpha$ is injective since the $A_i$ are flat. A simple diagram chase now shows that the left vertical map is surjective.



Thus the canonical map $Kotimes prod A_ito prod Kotimes A_i$ is surjective for any family of flat modules $(A_i)$. In particular, the canonical map $Kotimes R^Kto K^K$ is surjective, which we have seen above implies that $K$ is finitely generated.





By similar arguments, you can show that the canonical map $Motimes prod A_itoprod Motimes A_i$ is always a surjection iff $M$ is finitely generated. Indeed, the forward direction is already contained in the forward direction of the proof above, and the reverse direction is similar to the proof of the reverse direction above.






share|cite|improve this answer























  • Dear Eric Wofsey, thank you so much for this wonderful answer. I have a follow-up question: are there conditions on the $A_i$ that loosen the requirement on $M$? For instance $mathbb Q otimes_mathbb Z prod mathbb Z / p^i mathbb Z$ clearly does not distribute, while in some sense -- as you commented on Qiaochu Yuan's answer -- $mathbb Q otimes_mathbb Z prod mathbb Z$ does.
    – foaly
    Dec 28 '18 at 6:13












  • First of all, you absolutely should not think that $mathbb Q otimes_mathbb Z prod mathbb Z$ "distributes" in any reasonable sense. Yes, it is isomorphic to what you would get by distributing, but the isomorphism is highly noncanonical and probably can't even be proved to exist without the axiom of choice. It's essentially a total accident that they are isomorphic.
    – Eric Wofsey
    Dec 28 '18 at 6:53












  • Second, notice that the counterexamples constructed in my proof only ever actually use the case where $A_i=R$ for all $i$. What could be nicer than that? So, I doubt there is any interesting restriction you could put on the $A_i$ that would weaken the condition on $M$.
    – Eric Wofsey
    Dec 28 '18 at 6:55
















8














Here is a general criterion. (All tensor products in this answer are over $R$.)




Theorem: Let $R$ be a ring and let $M$ be a right $R$-module. Then the following are equivalent.




  1. The functor $Motimes -$ preserves products: that is, for every family $(A_i)$ of left $R$-modules, the canonical map $Motimesprod A_ito prod Motimes A_i$ is an isomorphism.


  2. $M$ is finitely presented.





Proof: First, suppose $M$ has a finite presentation $$R^mto R^nto Mto 0$$ and let $(A_i)$ be any family of left $R$-modules. We then get a commutative diagram
$$require{AMScd}
begin{CD}
R^motimes prod A_i @>>> R^notimes prod A_i @>>> Motimes prod A_i @>>> 0\
@VV{}V @VV{}V @VV{}V \
prod R^motimes A_i @>>> prod R^notimes A_i @>>> prod Motimes A_i @>>> 0
end{CD}$$
whose rows are exact. Now note that $R^motimes prod A_icong (prod A_i)^m$ and $prod R^motimes A_icong prod A_i^m$, and our vertical map between them is easily seen to be the canonical isomorphism which interchanges the products. So the left vertical map is an isomorphism and similarly so is the middle vertical map. By the five lemma, it follows that the right vertical map is an isomorphism, and thus $Motimes -$ preserves products.



Conversely, suppose $Motimes-$ preserves products. In particular, then, the canonical map $$varphi: Motimes R^Mto M^M$$ is an isomorphism. Considering the identity map $id:Mto M$ as an element of the product $M^M$, we have $$id=varphi(sum m_i otimes f_i)$$ for some finite collection of elements $m_iin M$ and $f_i:Mto R$. Evaluating both sides of this equation at an element $min M$ we find $$m=sum m_if_i(m).$$ Thus every element of $M$ is a linear combination of the elements $m_i$, so $M$ is finitely generated.



Now let $$0to Kto Fto Mto 0$$ be a presentation of $M$ with $F$ a finitely generated free module. To conclude $M$ is finitely presented, we must show that $K$ is finitely generated. Now let $(A_i)$ be any family of flat left $R$-modules and consider the commutative diagram
$$require{AMScd}
begin{CD}
Kotimes prod A_i @>>> Fotimes prod A_i @>>> Motimes prod A_i @>>> 0\
@VV{}V @VV{}V @VV{}V \
prod Kotimes A_i @>{alpha}>> prod Fotimes A_i @>>> prod Motimes A_i @>>> 0
end{CD}$$

which again has exact rows. The right vertical map is an isomorphism by hypothesis and the middle vertical map is an isomorphism since $F$ is finitely generated and free. Moreover, the map $alpha$ is injective since the $A_i$ are flat. A simple diagram chase now shows that the left vertical map is surjective.



Thus the canonical map $Kotimes prod A_ito prod Kotimes A_i$ is surjective for any family of flat modules $(A_i)$. In particular, the canonical map $Kotimes R^Kto K^K$ is surjective, which we have seen above implies that $K$ is finitely generated.





By similar arguments, you can show that the canonical map $Motimes prod A_itoprod Motimes A_i$ is always a surjection iff $M$ is finitely generated. Indeed, the forward direction is already contained in the forward direction of the proof above, and the reverse direction is similar to the proof of the reverse direction above.






share|cite|improve this answer























  • Dear Eric Wofsey, thank you so much for this wonderful answer. I have a follow-up question: are there conditions on the $A_i$ that loosen the requirement on $M$? For instance $mathbb Q otimes_mathbb Z prod mathbb Z / p^i mathbb Z$ clearly does not distribute, while in some sense -- as you commented on Qiaochu Yuan's answer -- $mathbb Q otimes_mathbb Z prod mathbb Z$ does.
    – foaly
    Dec 28 '18 at 6:13












  • First of all, you absolutely should not think that $mathbb Q otimes_mathbb Z prod mathbb Z$ "distributes" in any reasonable sense. Yes, it is isomorphic to what you would get by distributing, but the isomorphism is highly noncanonical and probably can't even be proved to exist without the axiom of choice. It's essentially a total accident that they are isomorphic.
    – Eric Wofsey
    Dec 28 '18 at 6:53












  • Second, notice that the counterexamples constructed in my proof only ever actually use the case where $A_i=R$ for all $i$. What could be nicer than that? So, I doubt there is any interesting restriction you could put on the $A_i$ that would weaken the condition on $M$.
    – Eric Wofsey
    Dec 28 '18 at 6:55














8












8








8






Here is a general criterion. (All tensor products in this answer are over $R$.)




Theorem: Let $R$ be a ring and let $M$ be a right $R$-module. Then the following are equivalent.




  1. The functor $Motimes -$ preserves products: that is, for every family $(A_i)$ of left $R$-modules, the canonical map $Motimesprod A_ito prod Motimes A_i$ is an isomorphism.


  2. $M$ is finitely presented.





Proof: First, suppose $M$ has a finite presentation $$R^mto R^nto Mto 0$$ and let $(A_i)$ be any family of left $R$-modules. We then get a commutative diagram
$$require{AMScd}
begin{CD}
R^motimes prod A_i @>>> R^notimes prod A_i @>>> Motimes prod A_i @>>> 0\
@VV{}V @VV{}V @VV{}V \
prod R^motimes A_i @>>> prod R^notimes A_i @>>> prod Motimes A_i @>>> 0
end{CD}$$
whose rows are exact. Now note that $R^motimes prod A_icong (prod A_i)^m$ and $prod R^motimes A_icong prod A_i^m$, and our vertical map between them is easily seen to be the canonical isomorphism which interchanges the products. So the left vertical map is an isomorphism and similarly so is the middle vertical map. By the five lemma, it follows that the right vertical map is an isomorphism, and thus $Motimes -$ preserves products.



Conversely, suppose $Motimes-$ preserves products. In particular, then, the canonical map $$varphi: Motimes R^Mto M^M$$ is an isomorphism. Considering the identity map $id:Mto M$ as an element of the product $M^M$, we have $$id=varphi(sum m_i otimes f_i)$$ for some finite collection of elements $m_iin M$ and $f_i:Mto R$. Evaluating both sides of this equation at an element $min M$ we find $$m=sum m_if_i(m).$$ Thus every element of $M$ is a linear combination of the elements $m_i$, so $M$ is finitely generated.



Now let $$0to Kto Fto Mto 0$$ be a presentation of $M$ with $F$ a finitely generated free module. To conclude $M$ is finitely presented, we must show that $K$ is finitely generated. Now let $(A_i)$ be any family of flat left $R$-modules and consider the commutative diagram
$$require{AMScd}
begin{CD}
Kotimes prod A_i @>>> Fotimes prod A_i @>>> Motimes prod A_i @>>> 0\
@VV{}V @VV{}V @VV{}V \
prod Kotimes A_i @>{alpha}>> prod Fotimes A_i @>>> prod Motimes A_i @>>> 0
end{CD}$$

which again has exact rows. The right vertical map is an isomorphism by hypothesis and the middle vertical map is an isomorphism since $F$ is finitely generated and free. Moreover, the map $alpha$ is injective since the $A_i$ are flat. A simple diagram chase now shows that the left vertical map is surjective.



Thus the canonical map $Kotimes prod A_ito prod Kotimes A_i$ is surjective for any family of flat modules $(A_i)$. In particular, the canonical map $Kotimes R^Kto K^K$ is surjective, which we have seen above implies that $K$ is finitely generated.





By similar arguments, you can show that the canonical map $Motimes prod A_itoprod Motimes A_i$ is always a surjection iff $M$ is finitely generated. Indeed, the forward direction is already contained in the forward direction of the proof above, and the reverse direction is similar to the proof of the reverse direction above.






share|cite|improve this answer














Here is a general criterion. (All tensor products in this answer are over $R$.)




Theorem: Let $R$ be a ring and let $M$ be a right $R$-module. Then the following are equivalent.




  1. The functor $Motimes -$ preserves products: that is, for every family $(A_i)$ of left $R$-modules, the canonical map $Motimesprod A_ito prod Motimes A_i$ is an isomorphism.


  2. $M$ is finitely presented.





Proof: First, suppose $M$ has a finite presentation $$R^mto R^nto Mto 0$$ and let $(A_i)$ be any family of left $R$-modules. We then get a commutative diagram
$$require{AMScd}
begin{CD}
R^motimes prod A_i @>>> R^notimes prod A_i @>>> Motimes prod A_i @>>> 0\
@VV{}V @VV{}V @VV{}V \
prod R^motimes A_i @>>> prod R^notimes A_i @>>> prod Motimes A_i @>>> 0
end{CD}$$
whose rows are exact. Now note that $R^motimes prod A_icong (prod A_i)^m$ and $prod R^motimes A_icong prod A_i^m$, and our vertical map between them is easily seen to be the canonical isomorphism which interchanges the products. So the left vertical map is an isomorphism and similarly so is the middle vertical map. By the five lemma, it follows that the right vertical map is an isomorphism, and thus $Motimes -$ preserves products.



Conversely, suppose $Motimes-$ preserves products. In particular, then, the canonical map $$varphi: Motimes R^Mto M^M$$ is an isomorphism. Considering the identity map $id:Mto M$ as an element of the product $M^M$, we have $$id=varphi(sum m_i otimes f_i)$$ for some finite collection of elements $m_iin M$ and $f_i:Mto R$. Evaluating both sides of this equation at an element $min M$ we find $$m=sum m_if_i(m).$$ Thus every element of $M$ is a linear combination of the elements $m_i$, so $M$ is finitely generated.



Now let $$0to Kto Fto Mto 0$$ be a presentation of $M$ with $F$ a finitely generated free module. To conclude $M$ is finitely presented, we must show that $K$ is finitely generated. Now let $(A_i)$ be any family of flat left $R$-modules and consider the commutative diagram
$$require{AMScd}
begin{CD}
Kotimes prod A_i @>>> Fotimes prod A_i @>>> Motimes prod A_i @>>> 0\
@VV{}V @VV{}V @VV{}V \
prod Kotimes A_i @>{alpha}>> prod Fotimes A_i @>>> prod Motimes A_i @>>> 0
end{CD}$$

which again has exact rows. The right vertical map is an isomorphism by hypothesis and the middle vertical map is an isomorphism since $F$ is finitely generated and free. Moreover, the map $alpha$ is injective since the $A_i$ are flat. A simple diagram chase now shows that the left vertical map is surjective.



Thus the canonical map $Kotimes prod A_ito prod Kotimes A_i$ is surjective for any family of flat modules $(A_i)$. In particular, the canonical map $Kotimes R^Kto K^K$ is surjective, which we have seen above implies that $K$ is finitely generated.





By similar arguments, you can show that the canonical map $Motimes prod A_itoprod Motimes A_i$ is always a surjection iff $M$ is finitely generated. Indeed, the forward direction is already contained in the forward direction of the proof above, and the reverse direction is similar to the proof of the reverse direction above.







share|cite|improve this answer














share|cite|improve this answer



share|cite|improve this answer








edited Dec 27 '18 at 6:36

























answered Dec 27 '18 at 6:00









Eric Wofsey

180k12205332




180k12205332












  • Dear Eric Wofsey, thank you so much for this wonderful answer. I have a follow-up question: are there conditions on the $A_i$ that loosen the requirement on $M$? For instance $mathbb Q otimes_mathbb Z prod mathbb Z / p^i mathbb Z$ clearly does not distribute, while in some sense -- as you commented on Qiaochu Yuan's answer -- $mathbb Q otimes_mathbb Z prod mathbb Z$ does.
    – foaly
    Dec 28 '18 at 6:13












  • First of all, you absolutely should not think that $mathbb Q otimes_mathbb Z prod mathbb Z$ "distributes" in any reasonable sense. Yes, it is isomorphic to what you would get by distributing, but the isomorphism is highly noncanonical and probably can't even be proved to exist without the axiom of choice. It's essentially a total accident that they are isomorphic.
    – Eric Wofsey
    Dec 28 '18 at 6:53












  • Second, notice that the counterexamples constructed in my proof only ever actually use the case where $A_i=R$ for all $i$. What could be nicer than that? So, I doubt there is any interesting restriction you could put on the $A_i$ that would weaken the condition on $M$.
    – Eric Wofsey
    Dec 28 '18 at 6:55


















  • Dear Eric Wofsey, thank you so much for this wonderful answer. I have a follow-up question: are there conditions on the $A_i$ that loosen the requirement on $M$? For instance $mathbb Q otimes_mathbb Z prod mathbb Z / p^i mathbb Z$ clearly does not distribute, while in some sense -- as you commented on Qiaochu Yuan's answer -- $mathbb Q otimes_mathbb Z prod mathbb Z$ does.
    – foaly
    Dec 28 '18 at 6:13












  • First of all, you absolutely should not think that $mathbb Q otimes_mathbb Z prod mathbb Z$ "distributes" in any reasonable sense. Yes, it is isomorphic to what you would get by distributing, but the isomorphism is highly noncanonical and probably can't even be proved to exist without the axiom of choice. It's essentially a total accident that they are isomorphic.
    – Eric Wofsey
    Dec 28 '18 at 6:53












  • Second, notice that the counterexamples constructed in my proof only ever actually use the case where $A_i=R$ for all $i$. What could be nicer than that? So, I doubt there is any interesting restriction you could put on the $A_i$ that would weaken the condition on $M$.
    – Eric Wofsey
    Dec 28 '18 at 6:55
















Dear Eric Wofsey, thank you so much for this wonderful answer. I have a follow-up question: are there conditions on the $A_i$ that loosen the requirement on $M$? For instance $mathbb Q otimes_mathbb Z prod mathbb Z / p^i mathbb Z$ clearly does not distribute, while in some sense -- as you commented on Qiaochu Yuan's answer -- $mathbb Q otimes_mathbb Z prod mathbb Z$ does.
– foaly
Dec 28 '18 at 6:13






Dear Eric Wofsey, thank you so much for this wonderful answer. I have a follow-up question: are there conditions on the $A_i$ that loosen the requirement on $M$? For instance $mathbb Q otimes_mathbb Z prod mathbb Z / p^i mathbb Z$ clearly does not distribute, while in some sense -- as you commented on Qiaochu Yuan's answer -- $mathbb Q otimes_mathbb Z prod mathbb Z$ does.
– foaly
Dec 28 '18 at 6:13














First of all, you absolutely should not think that $mathbb Q otimes_mathbb Z prod mathbb Z$ "distributes" in any reasonable sense. Yes, it is isomorphic to what you would get by distributing, but the isomorphism is highly noncanonical and probably can't even be proved to exist without the axiom of choice. It's essentially a total accident that they are isomorphic.
– Eric Wofsey
Dec 28 '18 at 6:53






First of all, you absolutely should not think that $mathbb Q otimes_mathbb Z prod mathbb Z$ "distributes" in any reasonable sense. Yes, it is isomorphic to what you would get by distributing, but the isomorphism is highly noncanonical and probably can't even be proved to exist without the axiom of choice. It's essentially a total accident that they are isomorphic.
– Eric Wofsey
Dec 28 '18 at 6:53














Second, notice that the counterexamples constructed in my proof only ever actually use the case where $A_i=R$ for all $i$. What could be nicer than that? So, I doubt there is any interesting restriction you could put on the $A_i$ that would weaken the condition on $M$.
– Eric Wofsey
Dec 28 '18 at 6:55




Second, notice that the counterexamples constructed in my proof only ever actually use the case where $A_i=R$ for all $i$. What could be nicer than that? So, I doubt there is any interesting restriction you could put on the $A_i$ that would weaken the condition on $M$.
– Eric Wofsey
Dec 28 '18 at 6:55











4















For instance, it would seem intuitive that $mathbb Q otimes_mathbb Z prod_mathbb N mathbb Z cong prod_mathbb N mathbb Q$. But is it?




It is not. The natural map from the LHS to the RHS fails to be surjective; its image is the subgroup of $prod mathbb{Q}$ consisting of those sequences whose terms have a common denominator, and hence does not include, for example, $prod frac{1}{n}$.



In general, for modules over a commutative ring $k$, the tensor product $M otimes_k (-)$ preserves infinite products if $M$ is finitely presented projective. I had thought at some point about whether it suffices for $M$ just to be finitely presented, but I don't remember what the conclusion was off the top of my head.






share|cite|improve this answer

















  • 4




    To be pedantic, those groups actually are isomorphic (though not by the natural map), since both are $mathbb{Q}$-vector spaces of dimension $mathfrak{c}$. Of course, the right question is whether the natural map is an isomorphism.
    – Eric Wofsey
    Dec 27 '18 at 5:11












  • Thank you for this answer, Qiaochu Yuan. Please allow me to ask some naive questions. As you point out, the canonical map is not an isomorphism. Nevertheless, as @EricWofsey points out, the two modules have some isomorphism. In what sense is the question after the natural map "the right question"? Also I am not familiar with the notation "dimension $mathfrak c$"; I assume it means countably infinite.
    – foaly
    Dec 28 '18 at 6:20












  • @foaly: "tensor product distributes over infinite direct products" is a question about the natural map being an isomorphism. Asking for there to be some arbitrary isomorphism is basically never the question you actually care about in practice.
    – Qiaochu Yuan
    Dec 28 '18 at 9:25










  • @QiaochuYuan it does not seem obvious to me why one would not be interested in the existence of some isomorphism. Can you elaborate?
    – foaly
    Dec 28 '18 at 9:29
















4















For instance, it would seem intuitive that $mathbb Q otimes_mathbb Z prod_mathbb N mathbb Z cong prod_mathbb N mathbb Q$. But is it?




It is not. The natural map from the LHS to the RHS fails to be surjective; its image is the subgroup of $prod mathbb{Q}$ consisting of those sequences whose terms have a common denominator, and hence does not include, for example, $prod frac{1}{n}$.



In general, for modules over a commutative ring $k$, the tensor product $M otimes_k (-)$ preserves infinite products if $M$ is finitely presented projective. I had thought at some point about whether it suffices for $M$ just to be finitely presented, but I don't remember what the conclusion was off the top of my head.






share|cite|improve this answer

















  • 4




    To be pedantic, those groups actually are isomorphic (though not by the natural map), since both are $mathbb{Q}$-vector spaces of dimension $mathfrak{c}$. Of course, the right question is whether the natural map is an isomorphism.
    – Eric Wofsey
    Dec 27 '18 at 5:11












  • Thank you for this answer, Qiaochu Yuan. Please allow me to ask some naive questions. As you point out, the canonical map is not an isomorphism. Nevertheless, as @EricWofsey points out, the two modules have some isomorphism. In what sense is the question after the natural map "the right question"? Also I am not familiar with the notation "dimension $mathfrak c$"; I assume it means countably infinite.
    – foaly
    Dec 28 '18 at 6:20












  • @foaly: "tensor product distributes over infinite direct products" is a question about the natural map being an isomorphism. Asking for there to be some arbitrary isomorphism is basically never the question you actually care about in practice.
    – Qiaochu Yuan
    Dec 28 '18 at 9:25










  • @QiaochuYuan it does not seem obvious to me why one would not be interested in the existence of some isomorphism. Can you elaborate?
    – foaly
    Dec 28 '18 at 9:29














4












4








4







For instance, it would seem intuitive that $mathbb Q otimes_mathbb Z prod_mathbb N mathbb Z cong prod_mathbb N mathbb Q$. But is it?




It is not. The natural map from the LHS to the RHS fails to be surjective; its image is the subgroup of $prod mathbb{Q}$ consisting of those sequences whose terms have a common denominator, and hence does not include, for example, $prod frac{1}{n}$.



In general, for modules over a commutative ring $k$, the tensor product $M otimes_k (-)$ preserves infinite products if $M$ is finitely presented projective. I had thought at some point about whether it suffices for $M$ just to be finitely presented, but I don't remember what the conclusion was off the top of my head.






share|cite|improve this answer













For instance, it would seem intuitive that $mathbb Q otimes_mathbb Z prod_mathbb N mathbb Z cong prod_mathbb N mathbb Q$. But is it?




It is not. The natural map from the LHS to the RHS fails to be surjective; its image is the subgroup of $prod mathbb{Q}$ consisting of those sequences whose terms have a common denominator, and hence does not include, for example, $prod frac{1}{n}$.



In general, for modules over a commutative ring $k$, the tensor product $M otimes_k (-)$ preserves infinite products if $M$ is finitely presented projective. I had thought at some point about whether it suffices for $M$ just to be finitely presented, but I don't remember what the conclusion was off the top of my head.







share|cite|improve this answer












share|cite|improve this answer



share|cite|improve this answer










answered Dec 27 '18 at 4:57









Qiaochu Yuan

277k32581919




277k32581919








  • 4




    To be pedantic, those groups actually are isomorphic (though not by the natural map), since both are $mathbb{Q}$-vector spaces of dimension $mathfrak{c}$. Of course, the right question is whether the natural map is an isomorphism.
    – Eric Wofsey
    Dec 27 '18 at 5:11












  • Thank you for this answer, Qiaochu Yuan. Please allow me to ask some naive questions. As you point out, the canonical map is not an isomorphism. Nevertheless, as @EricWofsey points out, the two modules have some isomorphism. In what sense is the question after the natural map "the right question"? Also I am not familiar with the notation "dimension $mathfrak c$"; I assume it means countably infinite.
    – foaly
    Dec 28 '18 at 6:20












  • @foaly: "tensor product distributes over infinite direct products" is a question about the natural map being an isomorphism. Asking for there to be some arbitrary isomorphism is basically never the question you actually care about in practice.
    – Qiaochu Yuan
    Dec 28 '18 at 9:25










  • @QiaochuYuan it does not seem obvious to me why one would not be interested in the existence of some isomorphism. Can you elaborate?
    – foaly
    Dec 28 '18 at 9:29














  • 4




    To be pedantic, those groups actually are isomorphic (though not by the natural map), since both are $mathbb{Q}$-vector spaces of dimension $mathfrak{c}$. Of course, the right question is whether the natural map is an isomorphism.
    – Eric Wofsey
    Dec 27 '18 at 5:11












  • Thank you for this answer, Qiaochu Yuan. Please allow me to ask some naive questions. As you point out, the canonical map is not an isomorphism. Nevertheless, as @EricWofsey points out, the two modules have some isomorphism. In what sense is the question after the natural map "the right question"? Also I am not familiar with the notation "dimension $mathfrak c$"; I assume it means countably infinite.
    – foaly
    Dec 28 '18 at 6:20












  • @foaly: "tensor product distributes over infinite direct products" is a question about the natural map being an isomorphism. Asking for there to be some arbitrary isomorphism is basically never the question you actually care about in practice.
    – Qiaochu Yuan
    Dec 28 '18 at 9:25










  • @QiaochuYuan it does not seem obvious to me why one would not be interested in the existence of some isomorphism. Can you elaborate?
    – foaly
    Dec 28 '18 at 9:29








4




4




To be pedantic, those groups actually are isomorphic (though not by the natural map), since both are $mathbb{Q}$-vector spaces of dimension $mathfrak{c}$. Of course, the right question is whether the natural map is an isomorphism.
– Eric Wofsey
Dec 27 '18 at 5:11






To be pedantic, those groups actually are isomorphic (though not by the natural map), since both are $mathbb{Q}$-vector spaces of dimension $mathfrak{c}$. Of course, the right question is whether the natural map is an isomorphism.
– Eric Wofsey
Dec 27 '18 at 5:11














Thank you for this answer, Qiaochu Yuan. Please allow me to ask some naive questions. As you point out, the canonical map is not an isomorphism. Nevertheless, as @EricWofsey points out, the two modules have some isomorphism. In what sense is the question after the natural map "the right question"? Also I am not familiar with the notation "dimension $mathfrak c$"; I assume it means countably infinite.
– foaly
Dec 28 '18 at 6:20






Thank you for this answer, Qiaochu Yuan. Please allow me to ask some naive questions. As you point out, the canonical map is not an isomorphism. Nevertheless, as @EricWofsey points out, the two modules have some isomorphism. In what sense is the question after the natural map "the right question"? Also I am not familiar with the notation "dimension $mathfrak c$"; I assume it means countably infinite.
– foaly
Dec 28 '18 at 6:20














@foaly: "tensor product distributes over infinite direct products" is a question about the natural map being an isomorphism. Asking for there to be some arbitrary isomorphism is basically never the question you actually care about in practice.
– Qiaochu Yuan
Dec 28 '18 at 9:25




@foaly: "tensor product distributes over infinite direct products" is a question about the natural map being an isomorphism. Asking for there to be some arbitrary isomorphism is basically never the question you actually care about in practice.
– Qiaochu Yuan
Dec 28 '18 at 9:25












@QiaochuYuan it does not seem obvious to me why one would not be interested in the existence of some isomorphism. Can you elaborate?
– foaly
Dec 28 '18 at 9:29




@QiaochuYuan it does not seem obvious to me why one would not be interested in the existence of some isomorphism. Can you elaborate?
– foaly
Dec 28 '18 at 9:29


















draft saved

draft discarded




















































Thanks for contributing an answer to Mathematics Stack Exchange!


  • Please be sure to answer the question. Provide details and share your research!

But avoid



  • Asking for help, clarification, or responding to other answers.

  • Making statements based on opinion; back them up with references or personal experience.


Use MathJax to format equations. MathJax reference.


To learn more, see our tips on writing great answers.





Some of your past answers have not been well-received, and you're in danger of being blocked from answering.


Please pay close attention to the following guidance:


  • Please be sure to answer the question. Provide details and share your research!

But avoid



  • Asking for help, clarification, or responding to other answers.

  • Making statements based on opinion; back them up with references or personal experience.


To learn more, see our tips on writing great answers.




draft saved


draft discarded














StackExchange.ready(
function () {
StackExchange.openid.initPostLogin('.new-post-login', 'https%3a%2f%2fmath.stackexchange.com%2fquestions%2f3053569%2fwhen-does-the-tensor-product-distribute-over-an-infinite-direct-product%23new-answer', 'question_page');
}
);

Post as a guest















Required, but never shown





















































Required, but never shown














Required, but never shown












Required, but never shown







Required, but never shown

































Required, but never shown














Required, but never shown












Required, but never shown







Required, but never shown







Popular posts from this blog

How did Captain America manage to do this?

迪纳利

南乌拉尔铁路局